Finding rate of change of moment of inertia tensor

In summary, the moment of inertia tensor is calculated by integrating the mass density of an object over its volume, taking into account the distance of each infinitesimal mass element from the axis of rotation. It is a measure of an object's resistance to rotational motion and plays a crucial role in determining its rotational dynamics. The rate of change of the moment of inertia tensor can be calculated by differentiating it with respect to time. It can change due to factors such as changes in mass distribution, shape or orientation, and external forces or torques. The moment of inertia tensor differs from the moment of inertia of a point mass as it considers the distribution of mass throughout an object and is represented as a 3x3 matrix rather than a scalar value.
  • #1
TadeusPrastowo
21
0

Homework Statement



The Wikipedia article on spatial rigid body dynamics derives the equation of motion [itex]\boldsymbol\tau = \boldsymbol\alpha + \boldsymbol\omega\times\boldsymbol\omega[/itex] from [itex]\sum_{i=1}^n \boldsymbol\Delta\mathbf{r}_i\times (m_i\mathbf{a}_i)[/itex].

But, there is another way to derive the same result from [itex]\frac{\text{d}\mathbf{L}}{\text{d}t} = \frac{\text{d}}{\text{d}t}\left(\boldsymbol\omega\right)[/itex]. Can it be derived by performing element-by-element derivation of the moment of inertia tensor?

Homework Equations



  • [tex]\frac{\text{d}}{\text{d}t}\left(\boldsymbol\omega\right) = \frac{\text{d}}{\text{d}t}\boldsymbol\omega + \frac{\text{d}\boldsymbol\omega}{\text{d}t}[/tex]
    [*][tex]\frac{\text{d}}{\text{d}t} = [\omega]_\times\, + \,[\omega]_\times[/tex] as shown in Point 5 of this article.


The Attempt at a Solution



[tex]\begin{align}
\frac{\text{d}}{\text{d}t}\left(\boldsymbol\omega\right) &= \frac{\text{d}}{\text{d}t}\boldsymbol\omega + \frac{\text{d}\boldsymbol\omega}{\text{d}t} \\
&= \frac{\text{d}}{\text{d}t}\boldsymbol\omega + \boldsymbol\alpha \\
\end{align}[/tex]

Now, I am focusing on solving [itex]\frac{\text{d}}{\text{d}t}[/itex] with the goal of obtaining [itex][\omega]_\times\, + \,[\omega]_\times[/itex].
I perform element-by-element derivation of the moment of inertia tensor as follows:

[tex]
\begin{align}
\frac{\text{d}}{\text{d}t}\left(\begin{bmatrix}
I_{1,1} & I_{1,2} & I_{1,3} \\
I_{2,1} & I_{2,2} & I_{2,3} \\
I_{3,1} & I_{3,2} & I_{3,3} \\
\end{bmatrix}\right) &= \begin{bmatrix}
\frac{\text{d}\,I_{1,1}}{\text{d}t} & \frac{\text{d}\,I_{1,2}}{\text{d}t} & \frac{\text{d}\,I_{1,3}}{\text{d}t} \\
\frac{\text{d}\,I_{2,1}}{\text{d}t} & \frac{\text{d}\,I_{2,2}}{\text{d}t} & \frac{\text{d}\,I_{2,3}}{\text{d}t} \\
\frac{\text{d}\,I_{3,1}}{\text{d}t} & \frac{\text{d}\,I_{3,2}}{\text{d}t} & \frac{\text{d}\,I_{3,3}}{\text{d}t} \\
\end{bmatrix}
\end{align}
[/tex]

where the moment of inertia tensor is as follows: [tex]I_{i,j} = \sum_p m_p\,\left(\delta_{i,j} \sum_k r_k^2 - r_i\,r_j\right)[/tex]

Then, I calculated the above one as follows:
[tex]\begin{align}
\frac{\text{d}I_{i,j}}{\text{d}t} &= \frac{\text{d}}{\text{d}t}\left(\sum_p m_p\,\left(\delta_{i,j} \sum_k r_k^2 - r_i\,r_j\right)\right) \\
&= \sum_p m_p \frac{\text{d}}{\text{d}t}\,\left(\delta_{i,j} \sum_k r_{p,k}^2 \right) - \frac{\text{d}}{\text{d}t}\,(r_{p,i}\,r_{p,j}) \\
&= \sum_p m_p \delta_{i,j} \frac{\text{d}}{\text{d}t}\, \left(\sum_k r_{p,k}^2\right) - \frac{\text{d}}{\text{d}t}\,(r_{p,i}\,r_{p,j}) \\
&= \sum_p m_p \delta_{i,j} \sum_k \frac{\text{d}}{\text{d}t}\,(r_{p,k}^2) - \frac{\text{d}}{\text{d}t}\,(r_{p,i}\,r_{p,j}) \\
&= \sum_p m_p \delta_{i,j} \sum_k \frac{\text{d}\,r_{p,k}^2}{\text{d}r_{p,k}} \frac{\text{d} \,r_{p,k}} { \text{d}t} - \frac{\text{d}}{\text{d}t}\,(r_{p,i}\,r_{p,j}) \\
&= \sum_p m_p \delta_{i,j} \sum_k 2\,r_{p,k}\,\omega_k - (\omega_i\,r_{p,j} + r_{p,i}\,\omega_j) \\
\frac{\text{d}\text{I}_{i,j}}{\text{d}t} &= \sum_p m_p \begin{bmatrix}
2\,r_{p,2}\,\omega_2 + 2\,r_{p,3}\,\omega_3 & -(r_{p,1}\,\omega_2 + r_{p,2}\,\omega_1) & -(r_{p,1}\,\omega_3 + r_{p,3}\,\omega_1) \\
-(r_{p,2}\,\omega_1 + r_{p,1}\,\omega_2) & 2\,r_{p,1}\,\omega_1 + 2\,r_{p,3}\,\omega_3 & -(r_{p,2}\,\omega_3 + r_{p,3}\,\omega_2) \\
-(r_{p,3}\,\omega_1 + r_{p,1}\,\omega_3) & -(r_{p,3}\,\omega_2 + r_{p,2}\,\omega_3) & 2\,r_{p,1}\omega_1 + 2\,r_{p,2}\,\omega_2
\end{bmatrix}
\end{align}
[/tex]

But, after trying some factorizations, I can't seem to get to the following:

[tex]\begin{align}
[\omega]_\times\,I_{\text{body}} + I_{\text{body}}\,[\omega]_\times &= \sum_p m_p \left( \begin{bmatrix}
0 & -\omega_3 & \omega_2 \\
\omega_3 & 0 & -\omega_1 \\
-\omega_2 & \omega_1 & 0
\end{bmatrix}\begin{bmatrix}
r_{p,2}^2 + r_{p,3}^2 & -r_{p,1} r_{p,2} & -r_{p,1} r_{p,3} \\
-r_{p,2} r_{p,1} & r_{p,1}^2 + r_{p,3}^2 & -r_{p,2} r_{p,3} \\
-r_{p,3} r_{p,1} & -r_{p,3} r_{p,2} & r_{p,1}^2 + r_{p,2}^2
\end{bmatrix} + \begin{bmatrix}
r_{p,2}^2 + r_{p,3}^2 & -r_{p,1} r_{p,2} & -r_{p,1} r_{p,3} \\
-r_{p,2} r_{p,1} & r_{p,1}^2 + r_{p,3}^2 & -r_{p,2} r_{p,3} \\
-r_{p,3} r_{p,1} & -r_{p,3} r_{p,2} & r_{p,1}^2 + r_{p,2}^2
\end{bmatrix}\begin{bmatrix}
0 & -\omega_3 & \omega_2 \\
\omega_3 & 0 & -\omega_1 \\
-\omega_2 & \omega_1 & 0
\end{bmatrix} \right) \\
&= \sum_p m_p \left( \begin{bmatrix}
r_{p,1}\,r_{p,2}\,\omega_3 - r_{p,1}\,r_{p,3}\,\omega_2 & \ldots \\
\vdots & \ddots \\
\end{bmatrix} + \begin{bmatrix}
-r_{p,1}\,r_{p,2}\,\omega_3 + r_{p,1}\,r_{p,3}\,\omega_2 & \ldots \\
\vdots & \ddots \\
\end{bmatrix} \right) \\
&= \sum_p m_p \begin{bmatrix}
0 & \ldots \\
\vdots & \ddots \\
\end{bmatrix} \ne \sum_p m_p \begin{bmatrix}
2\,r_{p,2}\,\omega_2 + 2\,r_{p,3}\,\omega_3 & \ldots \\
\vdots & \ddots \\
\end{bmatrix}
\end{align}
[/tex]

So, I should have made a mistake in the differentiation. But, where? And, how to proceed?

Or, is it because the definition of tensor cannot be used in performing the derivation? Why?
 
Physics news on Phys.org
  • #2
I think that you may have posted this in the wrong category. I may be wrong, though.
 
  • #3
TadeusPrastowo said:

Homework Statement



The Wikipedia article on spatial rigid body dynamics derives the equation of motion [itex]\boldsymbol\tau = \boldsymbol\alpha + \boldsymbol\omega\times\boldsymbol\omega[/itex] from [itex]\sum_{i=1}^n \boldsymbol\Delta\mathbf{r}_i\times (m_i\mathbf{a}_i)[/itex].

But, there is another way to derive the same result from [itex]\frac{\text{d}\mathbf{L}}{\text{d}t} = \frac{\text{d}}{\text{d}t}\left(\boldsymbol\omega\right)[/itex]. Can it be derived by performing element-by-element derivation of the moment of inertia tensor?

Homework Equations



  • [tex]\frac{\text{d}}{\text{d}t}\left(\boldsymbol\omega\right) = \frac{\text{d}}{\text{d}t}\boldsymbol\omega + \frac{\text{d}\boldsymbol\omega}{\text{d}t}[/tex]
    [*][tex]\frac{\text{d}}{\text{d}t} = [\omega]_\times\, + \,[\omega]_\times[/tex] as shown in Point 5 of this article.


The Attempt at a Solution



[tex]\begin{align}
\frac{\text{d}}{\text{d}t}\left(\boldsymbol\omega\right) &= \frac{\text{d}}{\text{d}t}\boldsymbol\omega + \frac{\text{d}\boldsymbol\omega}{\text{d}t} \\
&= \frac{\text{d}}{\text{d}t}\boldsymbol\omega + \boldsymbol\alpha \\
\end{align}[/tex]

Now, I am focusing on solving [itex]\frac{\text{d}}{\text{d}t}[/itex] with the goal of obtaining [itex][\omega]_\times\, + \,[\omega]_\times[/itex].
I perform element-by-element derivation of the moment of inertia tensor as follows:

[tex]
\begin{align}
\frac{\text{d}}{\text{d}t}\left(\begin{bmatrix}
I_{1,1} & I_{1,2} & I_{1,3} \\
I_{2,1} & I_{2,2} & I_{2,3} \\
I_{3,1} & I_{3,2} & I_{3,3} \\
\end{bmatrix}\right) &= \begin{bmatrix}
\frac{\text{d}\,I_{1,1}}{\text{d}t} & \frac{\text{d}\,I_{1,2}}{\text{d}t} & \frac{\text{d}\,I_{1,3}}{\text{d}t} \\
\frac{\text{d}\,I_{2,1}}{\text{d}t} & \frac{\text{d}\,I_{2,2}}{\text{d}t} & \frac{\text{d}\,I_{2,3}}{\text{d}t} \\
\frac{\text{d}\,I_{3,1}}{\text{d}t} & \frac{\text{d}\,I_{3,2}}{\text{d}t} & \frac{\text{d}\,I_{3,3}}{\text{d}t} \\
\end{bmatrix}
\end{align}
[/tex]

where the moment of inertia tensor is as follows: [tex]I_{i,j} = \sum_p m_p\,\left(\delta_{i,j} \sum_k r_k^2 - r_i\,r_j\right)[/tex]

Then, I calculated the above one as follows:
[tex]\begin{align}
\frac{\text{d}I_{i,j}}{\text{d}t} &= \frac{\text{d}}{\text{d}t}\left(\sum_p m_p\,\left(\delta_{i,j} \sum_k r_k^2 - r_i\,r_j\right)\right) \\
&= \sum_p m_p \frac{\text{d}}{\text{d}t}\,\left(\delta_{i,j} \sum_k r_{p,k}^2 \right) - \frac{\text{d}}{\text{d}t}\,(r_{p,i}\,r_{p,j}) \\
&= \sum_p m_p \delta_{i,j} \frac{\text{d}}{\text{d}t}\, \left(\sum_k r_{p,k}^2\right) - \frac{\text{d}}{\text{d}t}\,(r_{p,i}\,r_{p,j}) \\
&= \sum_p m_p \delta_{i,j} \sum_k \frac{\text{d}}{\text{d}t}\,(r_{p,k}^2) - \frac{\text{d}}{\text{d}t}\,(r_{p,i}\,r_{p,j}) \\
&= \sum_p m_p \delta_{i,j} \sum_k \frac{\text{d}\,r_{p,k}^2}{\text{d}r_{p,k}} \frac{\text{d} \,r_{p,k}} { \text{d}t} - \frac{\text{d}}{\text{d}t}\,(r_{p,i}\,r_{p,j}) \\
&= \sum_p m_p \delta_{i,j} \sum_k 2\,r_{p,k}\,\omega_k - (\omega_i\,r_{p,j} + r_{p,i}\,\omega_j) \\
\frac{\text{d}\text{I}_{i,j}}{\text{d}t} &= \sum_p m_p \begin{bmatrix}
2\,r_{p,2}\,\omega_2 + 2\,r_{p,3}\,\omega_3 & -(r_{p,1}\,\omega_2 + r_{p,2}\,\omega_1) & -(r_{p,1}\,\omega_3 + r_{p,3}\,\omega_1) \\
-(r_{p,2}\,\omega_1 + r_{p,1}\,\omega_2) & 2\,r_{p,1}\,\omega_1 + 2\,r_{p,3}\,\omega_3 & -(r_{p,2}\,\omega_3 + r_{p,3}\,\omega_2) \\
-(r_{p,3}\,\omega_1 + r_{p,1}\,\omega_3) & -(r_{p,3}\,\omega_2 + r_{p,2}\,\omega_3) & 2\,r_{p,1}\omega_1 + 2\,r_{p,2}\,\omega_2
\end{bmatrix}
\end{align}
[/tex]

But, after trying some factorizations, I can't seem to get to the following:

[tex]\begin{align}
[\omega]_\times\,I_{\text{body}} + I_{\text{body}}\,[\omega]_\times &= \sum_p m_p \left( \begin{bmatrix}
0 & -\omega_3 & \omega_2 \\
\omega_3 & 0 & -\omega_1 \\
-\omega_2 & \omega_1 & 0
\end{bmatrix}\begin{bmatrix}
r_{p,2}^2 + r_{p,3}^2 & -r_{p,1} r_{p,2} & -r_{p,1} r_{p,3} \\
-r_{p,2} r_{p,1} & r_{p,1}^2 + r_{p,3}^2 & -r_{p,2} r_{p,3} \\
-r_{p,3} r_{p,1} & -r_{p,3} r_{p,2} & r_{p,1}^2 + r_{p,2}^2
\end{bmatrix} + \begin{bmatrix}
r_{p,2}^2 + r_{p,3}^2 & -r_{p,1} r_{p,2} & -r_{p,1} r_{p,3} \\
-r_{p,2} r_{p,1} & r_{p,1}^2 + r_{p,3}^2 & -r_{p,2} r_{p,3} \\
-r_{p,3} r_{p,1} & -r_{p,3} r_{p,2} & r_{p,1}^2 + r_{p,2}^2
\end{bmatrix}\begin{bmatrix}
0 & -\omega_3 & \omega_2 \\
\omega_3 & 0 & -\omega_1 \\
-\omega_2 & \omega_1 & 0
\end{bmatrix} \right) \\
&= \sum_p m_p \left( \begin{bmatrix}
r_{p,1}\,r_{p,2}\,\omega_3 - r_{p,1}\,r_{p,3}\,\omega_2 & \ldots \\
\vdots & \ddots \\
\end{bmatrix} + \begin{bmatrix}
-r_{p,1}\,r_{p,2}\,\omega_3 + r_{p,1}\,r_{p,3}\,\omega_2 & \ldots \\
\vdots & \ddots \\
\end{bmatrix} \right) \\
&= \sum_p m_p \begin{bmatrix}
0 & \ldots \\
\vdots & \ddots \\
\end{bmatrix} \ne \sum_p m_p \begin{bmatrix}
2\,r_{p,2}\,\omega_2 + 2\,r_{p,3}\,\omega_3 & \ldots \\
\vdots & \ddots \\
\end{bmatrix}
\end{align}
[/tex]

So, I should have made a mistake in the differentiation. But, where? And, how to proceed?

Or, is it because the definition of tensor cannot be used in performing the derivation? Why?



A couple things. Firstly, damn that's a lot of latex. Secondly, I'm not sure exactly what it is you're trying to do. ##\omega## is a vector, and there for what you have must be a cross product, but its ##\omega \times \omega## which is always 0, seeing as how omega is parallel to itself.

Next thing, I'm not following the notation very well. You're cutting you're object up into 9 chunks and approximating it? And ##m_{p}## is the mass of each chunk? What is ##\delta_{i, j}## ? Is thisrepresentative of the fact that you're trying to use a delta epsilon statement for your approximation?
when you sum over k, what are you actually summing over? The same with p.
 
  • #4
I'm going to go to bed for tonight. I'm also going to blame my phone for the "there for".
XD
I'll check this again tomorrow.
Good night all
 
  • #5
No, it is not [itex]\boldsymbol\omega \times \boldsymbol\omega[/itex] but [itex]\boldsymbol\omega \times \boldsymbol\omega[/itex] where [itex][/itex] is a second-order tensor, not a scalar. To be exact, we are dealing with the product of matrices [itex][\omega]_\times \left( [\omega]\right)[/itex]. See Wikipedia article on cross-product as a matrix.

I think mafagafo may be correct that my post is in the wrong category. But, I don't see any facility to move this post to another category. Let me see... I will re-post if I can't find one.
 
  • #6
I would try to replace ##\omega_i## with ##(\omega \times r)_i## since dr/dt=##\omega \times r##
 
  • Like
Likes 1 person
  • #7
[tex]\begin{align}
\frac{\text{d}I_{i,j}}{\text{d}t} &= \frac{\text{d}}{\text{d}t}\left(\sum_p m_p\,\left(\delta_{i,j} \sum_k r_k^2 - r_i\,r_j\right)\right) \\
&= \sum_p m_p \frac{\text{d}}{\text{d}t}\,\left(\delta_{i,j} \sum_k r_{p,k}^2 \right) - \frac{\text{d}}{\text{d}t}\,(r_{p,i}\,r_{p,j}) \\
&= \sum_p m_p \delta_{i,j} \frac{\text{d}}{\text{d}t}\, \left(\sum_k r_{p,k}^2\right) - \frac{\text{d}}{\text{d}t}\,(r_{p,i}\,r_{p,j}) \\
&= \sum_p m_p \delta_{i,j} \sum_k \frac{\text{d}}{\text{d}t}\,(r_{p,k}^2) - \frac{\text{d}}{\text{d}t}\,(r_{p,i}\,r_{p,j}) \\
&= \sum_p m_p \delta_{i,j} \sum_k \frac{\text{d}\,r_{p,k}^2}{\text{d}r_{p,k}} \frac{\text{d} \,r_{p,k}} { \text{d}t} - \frac{\text{d}}{\text{d}t}\,(r_{p,i}\,r_{p,j}) \\
&= \sum_p m_p \delta_{i,j} \sum_k 2\,r_{p,k}\,\pmb{(\omega\,\times\,r_p)_k} - (\pmb{(\omega\,\times\,r_p)_i}\,r_{p,j} + r_{p,i}\,\pmb{(\omega\,\times\,r_p)_j})
\end{align}
[/tex]
 
  • #8
I have tried to continue the derivation after correcting the dr/dt as you pointed out.

I did [itex](\boldsymbol\omega \times \mathbf{r}_p)_k[/itex] by calculating [itex]\boldsymbol\omega \times \mathbf{r}_p[/itex] first as matrix multiplication [itex][\omega]_\times [r_p][/itex] and then taking the k-element.

Then, I continued the derivation up to the result of [itex]\frac{d}{dt}\boldsymbol\omega[/itex] to check if the result would equal to [itex][\omega]_\times (\boldsymbol\omega)[/itex].

Unfortunately, they differ by two or three terms based on my pencil & paper calculation.

So, it may be that I make another mistake on the paper. But, it might also be that the derivation cannot go that way (i.e., tensor cannot be derived in the way I suggested).

In the end, I decided to be done with tensor derivation and went another route by performing the calculation starting from the definition of angular momentum [itex]\mathbf{L}_p = \mathbf{r}_p \times \mathbf{p}_p[/itex] as follows:
[tex]\begin{align}
\frac{\text{d}\mathbf{L}}{\text{d}t} &= \frac{\text{d}}{\text{d}t} \left(\sum_p \mathbf{r}_p \times \mathbf{p}_p \right) \\
&= \frac{\text{d}}{\text{d}t} \left(\sum_p \mathbf{r}_p \times (m_p \mathbf{v}_p) \right) \\
&= \frac{\text{d}}{\text{d}t} \left(\sum_p m_p \left( \mathbf{r}_p \times (\boldsymbol\omega \times \mathbf{r}_p) \right)\right)
\end{align}[/tex]

By that route, I can arrive at [itex]\boldsymbol\alpha + \boldsymbol\omega \times ( \boldsymbol\omega)[/itex] using the derivation property of cross product.

So, I guess tensor derivation cannot be performed element-by-element like what I suggested.
 
Last edited:

1. How do you calculate the moment of inertia tensor?

The moment of inertia tensor is calculated by integrating the mass density of an object over its volume, taking into account the distance of each infinitesimal mass element from the axis of rotation. This can be represented mathematically as I = ∫r^2 dm, where r is the distance from the axis of rotation and dm is the mass element.

2. What is the significance of the moment of inertia tensor?

The moment of inertia tensor is a measure of an object's resistance to rotational motion. It plays a crucial role in determining the rotational dynamics of an object, such as its angular momentum and angular acceleration.

3. How can the rate of change of moment of inertia tensor be calculated?

The rate of change of moment of inertia tensor can be calculated by differentiating the moment of inertia tensor with respect to time. This can be represented mathematically as dI/dt = ∫r^2 d(dm/dt), where dm/dt is the rate of change of mass element with respect to time.

4. What factors can cause the moment of inertia tensor to change?

The moment of inertia tensor can change due to factors such as changes in the mass distribution of an object, changes in the shape or orientation of an object, and external forces or torques acting on an object.

5. How does the moment of inertia tensor differ from the moment of inertia of a point mass?

The moment of inertia tensor takes into account the distribution of mass throughout an object, while the moment of inertia of a point mass only considers the mass of a single point. The moment of inertia tensor is a 3x3 matrix, while the moment of inertia of a point mass is a scalar value.

Similar threads

  • Advanced Physics Homework Help
Replies
11
Views
1K
Replies
1
Views
383
  • Advanced Physics Homework Help
Replies
2
Views
824
  • Advanced Physics Homework Help
Replies
1
Views
1K
  • Advanced Physics Homework Help
Replies
1
Views
2K
  • Advanced Physics Homework Help
Replies
1
Views
1K
  • Advanced Physics Homework Help
Replies
1
Views
1K
  • Engineering and Comp Sci Homework Help
Replies
0
Views
523
  • Advanced Physics Homework Help
Replies
6
Views
1K
  • Advanced Physics Homework Help
Replies
6
Views
3K
Back
Top